The Super Discount store is having a sale and all the clearance items are %40 off. Which of the following are correct ways to find the price of an item that is $25.00 normally ?

Answers

Answer 1

The normal price is $25, we need to find the 40% of $25.

40% = 0.4

So if we multiply $25 by 0.4


Related Questions

If f(x) = -x² - 2x, what is f(-2)?

Answers

Answer: 0

Step-by-step explanation:

f(-2)= -(-2)^2-2(-2)

= -4+4=0

My dog got hurt and needed surgery,so I had to use my credit card to pay the vet bill. His surgery costed me $5,323.21. If my monthly interest rate is 1.42%, how much is my finance charge for the first billing cycle?

Answers

Answer:

$75.59.

Explanation:

• Cost of the surgery = $5,323.21

,

• Monthly interest rate = 1.42%

A finance charge is a fee charged for the use of a credit card. A billing cycle is usually between 28 to 31 days, i.e. a month.

To find the finance charge, multiply the interest rate by the cost of surgery.

[tex]\begin{gathered} \text{Fnance Charge}=1.42\%\text{ of \$}5,323.21 \\ =\frac{1.42}{100}\times5,323.21 \\ =\$75.59 \end{gathered}[/tex]

The finance charge for the first billing cycle is $75.59.

Can you help me figure out how to find the original radican ??? I have no clue how to do so

Answers

So we have:

[tex]-3a^5b^2\sqrt[3]{a^2c}[/tex]

And we want to knowthe original before simplification, that is, before evaluating the interior part of the root.

So, we need to figure a way to put the part outside of the root back in.

Taking the cubic root of a number is the same as dividing its exponent by 3, because:

[tex]\sqrt[3]{a^n}=a^{\frac{n}{3}}[/tex]

So, thinking in the other direction, we need to multiply the exponents by 3 before taking it back to the inside of the cubic root:

[tex]a^k=a^{\frac{3k}{3}}=\sqrt[3]{a^{3k}}[/tex]

So, the b part have a 2 in the exponent, so we can multiply it by 3 to get 6:

[tex]\begin{gathered} b^2=b^{\frac{3\cdot2}{3}}=\sqrt[3]{b^{3\cdot2}}=\sqrt[3]{b^6} \\ -3a^5b^2\sqrt[3]{a^2c}=-3a^5\sqrt[3]{b^6}\sqrt[3]{a^2b^{3\cdot2}c}=-3a^5\sqrt[3]{a^2b^6c} \end{gathered}[/tex]

The a part have a 5 in the exponent, so we will get 15:

[tex]\begin{gathered} a^5=a^{\frac{3\cdot5}{3}}=\sqrt[3]{a^{3\cdot5}}=\sqrt[3]{a^{15}} \\ -3a^5\sqrt[3]{a^2b^6c}=-3\sqrt[3]{a^{15}}\sqrt[3]{a^2^{}b^6c}=-3\sqrt[3]{a^2a^{15}b^6c} \end{gathered}[/tex]

Now, since we have and a¹⁵, we can add their exponents:

[tex]\begin{gathered} a^2a^{15}=a^{17} \\ -3\sqrt[3]{a^2a^{15}b^6c}=-3^{}\sqrt[3]{a^{17}b^6c} \end{gathered}[/tex]

Now, the -3 have an exponent of 1, so:

[tex]\begin{gathered} -3=(-3)^1=(-3)^{\frac{3\cdot1}{3}}=\sqrt[3]{(-3)^{3\cdot1}}=\sqrt[3]{(-3)^3}=\sqrt[3]{-27} \\ -3^{}\sqrt[3]{a^{17}b^6c}=\sqrt[3]{-27}^{}\sqrt[3]{a^{17}b^6c}=^{}\sqrt[3]{-27a^{17}b^6c} \end{gathered}[/tex]

Thus, we have, in the end:

[tex]^{}\sqrt[3]{-27a^{17}b^6c}=-3a^5b^2^{}\sqrt[3]{a^2^{}c}[/tex]

Solve for 3x/2 -4 = 16what does x equal??

Answers

The given equation is expressed as

3x/2 - 4 = 16

The first step is to multiply both sides of the equation by 2. It becomes

3x/2 * 2 - 4 * 2 = 16 * 2

3x - 8 = 32

3x = 32 + 8

3x = 40

x = 40/3

x = 13.33

One equation from a system of two linear equations is graphed on the coordinate grid. 51 46 5 4 3 2 1 6 x -1 -21 The second equation in the system of linear equations has a slope of 3 and passes through the point (2,-5). What is the solution to the system of equations? th

Answers

First, we need to find the equation for the two equations.

The equation graphed has a y-intercept of 3 and a slope of

[tex]m=\frac{-6}{3}=-3[/tex]

therefore, the equation of the line is

[tex]\boxed{y=-\frac{1}{2}x+3.}[/tex]

For the second equation, we know what it has a slope of 3; therefore it can be written as

[tex]y=3x+b[/tex]

Now, we also know that this equation passes through the point y = -5, x = 2; therefore,

[tex]-5=3(2)+b[/tex]

which gives

[tex]-5=6+b[/tex][tex]b=-11[/tex]

Hence, the equation of the line is

[tex]\boxed{y=3x-11}[/tex]

Now we have the equations

[tex]\begin{gathered} y=-\frac{1}{2}x+3 \\ y=3x-11 \end{gathered}[/tex]

equating them gives

[tex]-\frac{1}{2}x+3=3x-11[/tex]

adding 11 to both sides gives

[tex]-\frac{1}{2}x+14=3x[/tex]

adding 1/2 x to both sides gives

[tex]14=\frac{7}{2}x[/tex]

Finally, dividing both sides by 7/2 gives

[tex]\boxed{x=4\text{.}}[/tex]

The corresponding value of y is found by substituting the above value into one of the equations

[tex]y=-\frac{1}{2}(4)+3[/tex][tex]y=1[/tex]

Hence, the solution to the system is

[tex](4,1)_{}[/tex]

There are two buildings beside a park.
The first building is 165 3/4 ft tall, and
the second building is 114 1/4 ft tall.
By rounding to the nearest whole number,
estimate the difference between the
heights of the buildings.

Answers

The difference of the height of the buildings is 51.5 for the first building is 165 3/4 feet tall, and the second building is 114 1/4 feet tall.

Given that,

There are two building side of a park.

The first building is 165 3/4 feet tall, and the second building is 114 1/4 feet tall.

We have to find the difference of the height of the buildings.

We have to subtract the first building is 165 3/4 feet tall, and the second building is 114 1/4 feet tall.

165 3/4- 114 1/4

660+3/4- 456+1/4

663/4-457/4

165.75-114.25

51.5

Therefore, The difference of the height of the buildings is 51.5 for the first building is 165 3/4 feet tall, and the second building is 114 1/4 feet tall.

To learn more about height visit: https://brainly.com/question/10726356

#SPJ9

Factor the polynomial completely.X^2+x+1

Answers

[tex]x(x+1)+1[/tex]

1) Examining the expression below, we can group the first and the second term:

[tex]\begin{gathered} x^2+x+1 \\ x(x+1)+1 \\ \end{gathered}[/tex]

Note that there is no way beyond this point. So we could not factor beyond this point.

Solve the inequality algebraically. Express your answer using set notation or interval notation. l x-8l greater than or equal to 4. Rewrite the inequality without the absolute values.

Answers

To rewrite the inequality:

[tex]\lvert{x-8}\rvert\ge4[/tex]

we need to remember that:

[tex]\lvert{x}\rvert\ge a\text{ is equivalent to }x\ge a\text{ or }x\leq-a[/tex]

Then in this case we have:

[tex]\begin{gathered} \lvert{x-8}\rvert\ge4 \\ \text{ Is equivalent to:} \\ x-8\ge4\text{ or }x-8\leq-4 \end{gathered}[/tex]

Therefore, we can rewrite the inequality as:

[tex]x-8\leq-4\text{ or }x-8\ge4[/tex]

Once we have it written in this form we can solve it:

[tex]\begin{gathered} x-8\leq-4\text{ or }x-8\ge4 \\ x\leq-4+8\text{ or }x\ge4+8 \\ x\leq4\text{ or }x\ge12 \end{gathered}[/tex]

Therefore, the solution set of the inequality is:

[tex](-\infty,4\rbrack\cup\lbrack12,\infty)[/tex]

Write a coordinate proof of the following theorem:"If a quadrilateral is a kite, then its diagonals are perpendicular."(image attached)thank you ! :)

Answers

Explanations:

Given the following coordinate points from the kite

[tex]\begin{gathered} W=(a,4b) \\ X=(2a,b) \\ Y=(a,0) \\ Z=(0,b) \end{gathered}[/tex]

For the diagonals to be perpendicular the product of the distance WY and XZ must be zero that is;

[tex]\vec{WY}\cdot\vec{XZ}=0[/tex]

Determine the coordinate point WY

[tex]\begin{gathered} \vec{WY}=[(a-a,4b-0)] \\ \vec{WY}=(0,4b) \end{gathered}[/tex]

Determine the coordinate point XZ

[tex]\begin{gathered} \vec{XZ}=[(2a-0),(b-b)] \\ \vec{XZ}=(2a,0) \end{gathered}[/tex]

Take the dot product of the coordinates

[tex]\begin{gathered} \vec{WY}\cdot\vec{XZ}=(0,4b)\cdot(2a,0) \\ \vec{WY}\cdot\vec{XZ}=[(0)(2a)+(4b)(0))] \\ \vec{WY}\cdot\vec{XZ}=(0,0)=\vec{0} \end{gathered}[/tex]

Since the dot product of the coordinates is a zero vector, hence its diagonals are perpendicular.

Write the equation of the circle:center at (5, - 2) , passes through (4, 0)

Answers

the equation of the circle is

(x-h)^2 + (y-k)^2 = r^2

if we replace the terms

(4-5)^2 + (0-(-2))^2 = r^2

Now, we can fin the radius if we solve the previous equation

( - 1 )^2 + ( 2 )^2 =r^2

1 + 4 = r^2

5 = r^2

r = SQRT(5)

Now, since we already know r, we can replace it in the circle equation to obtain the result

so, (x-h)^2 + (y-k)^2 = r^2

iquals to, (x-5)^2 + (y+2)^2 = 5

I NEED HELP QUICKLY ITS DUE 8PM AND I HAVE OTHER HOMEWORK TO DO.

Answers

Answer:

$5625

Explanation:

The equation for your earning y = 150x - x² is the equation of a parabola, so the maximum point of the parabola has a coordinate x equal to -b/2a

Where b is the number beside the x and a is the number beside the x²

In this case, a is -1 and b is 150, so the x-coordinate of the maximum is:

[tex]x=\frac{-b}{2a}=\frac{-150}{2(-1)}=\frac{-150}{-2}=75[/tex]

With the value of x, we can calculate the value of y, so:

y = 150x - x²

y = 150(75) - (75)²

y = 11250 - 5625

y = 5625

Therefore, the maximum amount that you can earn is $5625.

the point M 6,-4 is reflected over the y-axis. what are the cordnates of the resulting point, M?

Answers

A reflection over the Y-axis is given by:

[tex](x,y)\rightarrow(-x,y)[/tex]

Substitute for (x,y)=(6,-4):

[tex](6,-4)\rightarrow(-6,-4)[/tex]

Therefore, the new coordinates of the point after a reflection over the Y-axis, are:

[tex](-6,-4)[/tex]

Find the perimeter of the shaded region of this composite figure .You can use 3.14 for pi.llAlso round the answer to the nearest hundreth.

Answers

ANSWER

18.58 m

EXPLANATION

We need to find the perimeter of the shaded region of the figure.

The figure is made up of a rectangle with the cut out of a semi-circle, so, to find the perimeter, we will subtract the perimeter of the semi-circle (without the diameter) from that of the rectangle.

The perimeter of the rectangle is:

P = 2(L + B)

where L = length = 8 m

B = breadth = 6 m

So, the perimeter of the rectangle is:

P = 2(8 + 6) = 2 * 14

P = 28 m

The perimeter (circumference) of the semi-circle (without the diameter) is:

C = π * R

where R = radius of the semicircle

The diameter is 6 m, so the radius is:

R = D / 2 = 6 / 2 = 3 m

So, the circumference of the semicircle is:

C = 3.14 * 3

C = 9.42 m

So, the perimeter of the composite figure is:

P = 28 - 9.42

P = 18.58 m

That is the answer.

what is the quotient of the complex numbers below 3 + 2i / 1 - 5i

Answers

[tex]\frac{3+2i}{1-5i}[/tex]

Take the conjugate of the denominator, use it to multiply the numerator and the denominator

That is;

[tex]undefined[/tex]

Answer:

Step-by-step explanation:

A rocket is launched by Team Flash from the ground on Earth-73. The rocket passes a sensor at a height of5760 feet after 8 seconds and lands back on Earth-73 after 53 seconds.Write an equation for the height of the rocket, h, in feet as a function of the number of seconds, t, since therocket was launched.Round to 3 decimal places as needed.After how many seconds will the rocket reach its maximum height?Round to 3 decimal places as needed.What is the maximum height in feet that the rocket reaches?Round to 3 decimal places as needed.

Answers

We know two points of the trajectory of the rocket:

1) A height of 5760 ft at time t=8 seconds after launch.

2) A height of 0 ft (landing) at time t=53 seconds after launch.

We also know that the initial position was a height of 0 ft at t=0 seconds.

So we have 3 points to write the equation, that will be a quadratic equation for this kind of trayectory.

As we know we have roots at t=0 and t=53, we can start writing it as:

[tex]h(t)=a(t-0)(t-53)=at(t-53)[/tex]

We have one point left, (t,h) = (8, 5760), to find the parameter "a". We can replace t and y in the equation and solve as:

[tex]\begin{gathered} h(t)=at(t-53) \\ 5760=a\cdot8\cdot(8-53) \\ 5760=a\cdot8\cdot(-45) \\ 5760=a\cdot(-360) \\ a=\frac{5760}{-360} \\ a=-16 \end{gathered}[/tex]

Then we can write the equation as:

[tex]h=-16t(t-53)=-16t^2+848t[/tex]

We can graph it as:

In this kind of trajectories, the maximum height is reached halfway between the launch and the landing.

For any function, we can find the maximum of minimums deriving the function and equal it to 0. We will do it for this function:

[tex]\begin{gathered} \frac{dh}{dt}=-16(2t)+848=0 \\ -32t+848=0 \\ 32t=848 \\ t=\frac{848}{32} \\ t=26.5 \end{gathered}[/tex]

The maximum height is reached at time t=26.5 seconds.

Now we can calculate the height at t=26.5 seconds, the maximum height, as:

[tex]\begin{gathered} h(26.5)=-16(26.5)^2+848(26.5) \\ h(26.5)=-16\cdot702.25+22472 \\ h(26.5)=-11236+22472 \\ h(26.5)=11236 \end{gathered}[/tex]

Answer:

a) The equation is h(t) = -16t²+848t

b) The maximum height is reached at time t=26.5 seconds.

c) The maximum height is 11236 ft.

HELP. i am so confused. the question is in the picture

Answers

1) If we consider that y=f(2x) is a transformed version of y=f(x) then we can set a t-table and plug for the given point (16,8) the x-coordinate x=16

[tex]\begin{gathered} (16,8)-\longrightarrow f(x)-->y=\frac{1}{2}x \\ \end{gathered}[/tex]

Since the new function, f(2x) requires us to divide the input by 2 to compensate

how do I multeply Fractions

Answers

Multiplying simultaneously numerator by numerator and denominator by denominator.

You can multiply fractions, multiplying simultaneously numerator by numerator and denominator by denominator.

2) Notice that whenever possible, we must simplify it to the lowest possible fraction.

Can u please help me with This am trying to study but can’t get it

Answers

Given:

Following Matrices are given.

[tex]A=\begin{bmatrix}{2} & {1} \\ {3} & {4}\end{bmatrix},B=[\text{ }5\text{ 4 \rbrack, C=}\begin{bmatrix}{4} & {1} & {6} \\ {} & {} & {} \\ {5} & {2} & {7}\end{bmatrix}[/tex]

Find:

we have to find which Matrix multiplication can be defined.

Explanation:

For Matrix multiplication, the number of columns of first Matrix should be equal to the number of rows of the second Matrix.

Therefore, the following Matrix multiplication can be defined

BC,Because number of columns of B is 2 and number of rows of C is 2.

AC,Because number of columns of A is 2 and number of rows of C is 2.

BA,Because number of columns of B is 2 and number of rows of A is 2.

Therefore, the multiplications BC,AC,BA can be defined.

Solve 431 ÷ 3 on paper. You'll see that there is a remainder.
What digit in the ones place would give us no remainder?

Answers

Answer:

2 in the ones place.

Step-by-step explanation:

431/3 won't work because 4+3+1=NOT multiple of 3.

The closest number in the ones place that will make it an integer is 2, because 4+3+2=multiple of 3

Answer:

2, 5, or 8.

Step-by-step explanation:

There is actually a trick to this one.

---

If the digits in a number add up to a multiple of 3, then the whole number is divisible by 3.

For example:

843

Add the digits:

8+4+3=15

You could add the digits again:

1+5=6

Six is a multiple of 3, so 843 is a multiple of 3.

---

Now your number was 431.

4+3+1=8

8 is not divisible by 3, so 431 is not divisible by 3.

432, 435, and 438 would work in this situation.

PLEASE MARK AS BRANLIEST!!

3. Which expression is equivalent to 3(x-2) + Zx?A. -XB. 3xC. 5X-2D. 5X-6marios The cost of

Answers

Given an expression below :

[tex]3(x-2)+2x[/tex]

The expression can be solved by :

Step 1: Opening the bracket

[tex]\begin{gathered} 3(x-2)+2x \\ 3x-6+2x \end{gathered}[/tex]

Step 2: Collect like terms

[tex]\begin{gathered} 3x-6+2x \\ 3x+2x-6 \\ 5x-6 \end{gathered}[/tex]

Therefore the correct answer for the expression is 5x - 6

Hence the correct value is Option D

Solve the equation 42+7c - 5 = 0 using the quadratic formula

Answers

The equation is:

[tex]4c^2+7c-5=0[/tex]

so we can use the cuadratic equation so:

[tex]\begin{gathered} c=\frac{-b\pm\sqrt[]{b^2-4(a)(c^{\prime})}}{2(a)} \\ \text{where} \\ a=4 \\ b=7 \\ c^{\prime}=-5 \end{gathered}[/tex]

So if we replace in the equation we will have:

[tex]c=\frac{-7\pm\sqrt[]{7^2-4(4)(-5)}}{2(4)}[/tex]

So we simplify to solve the problem so:

[tex]c=\frac{-7\pm\sqrt[]{129}}{8}[/tex]

jenelle and hadiya went to lunch,the bill,before sales before sales tax and tip,was 37.50.a sales tax of 8% was added.they added an 18% tip on the amount after the tax was added.a)what was the amount,in dollars,of the sales tax.b)what was the total amount they paid,including tax and tip.

Answers

$37.50

tax = 8%

tip = 18%

a) 37.5 ------------------ 100%

x ------------------- 8%

x = (8 x 37.5) / 100

x = 3

Tax = $3

b) Money of lunch plus tax = $40.5

40.5 -------------------- 100

x --------------------- 18

x = (18 x 40.5) / 100

x = 7.29

Total amount paid = 7.29 + 40.5

= $ 47.79

Lakshmi bought 7 books for a total of 56 rupees how much would see pay for just three books? 56 rupees Indian money

Answers

To find how much would be paid for 3 books, follow the steps below.

Step 01: Find the price of one book.

Let's say the price of one book is x.

Then, the price of 7 books is 7 times x, which is 56 rupes.

[tex]7x=56[/tex]

To find x, let's divide both sides by 7:

[tex]\begin{gathered} \frac{7x}{7}=\frac{56}{7} \\ 1x=8 \\ x=8 \end{gathered}[/tex]

So, the price of one book is 8.

Step 02: Find the price of 3 books.

If the price of one book is 8, the price of 3 books (P) will be 3 times 8:

[tex]\begin{gathered} P=3\cdot8 \\ P=24 \end{gathered}[/tex]

Answer: It would be paid 24 rupees for 3 books.

Evan is going to the 50th state fair this weekend. It costs $10 to enterand each ride is $2. How much will it cost Evan to go to the fair and ride 5rides? **Don't forget the initial cost.**( hint: determine the equation first y =X + and then plug in 5 for x) *

Answers

From the question, we are given the following;

Cost of entering the state fair = $10

Amount of each ride = $2

For us to determine the amount it will cost Evan to go to the fair and ride 5 rides, the equation y = $10 + 2x will be used where;

x is the total ride taken = 5 rides

y is the amount it cost evan to enter the state fair and ride 5 rides

Substitute x = 5 into the equation and get y;

y = $10 + 2x

y = $10 + $2(5)

y = $10 + $10

y = $20

Hence it will cost Evans $20 to go to the fair and ride 5 rides

There is a population of 405,000 bacteria in a colony. If the number of bacteria doubles every 44 hours, what will the population be 176 hours from now?

Answers

Since the population doubles every 44 hours, it can be modeled using an exponential equation as follows:

[tex]P(t)=405,000\times2^{\frac{t}{44}}[/tex]

Where t is the time since the population was 405,000 measured in hours.

Replace t=176 to find the population after 176 hours:

[tex]\begin{gathered} P(176)=405,000\times2^{\frac{176}{44}} \\ =405,000\times2^4 \\ =405,000\times16 \\ =6,480,000 \end{gathered}[/tex]

Therefore, the population after 176 hours will be 6,480,000

Find the cost for each pound of jelly beans and each pound of almonds

Answers

Let 'x' represent the cost for each pound of jelly beans.

Let 'y' represent the cost for each pound of almonds.

For the first statement, the mathematical expression is

[tex]\begin{gathered} 9x+7y=37\ldots\ldots1 \\ \end{gathered}[/tex]

For the second statement, the mathematical expression is,

[tex]3x+5y=17\ldots\ldots2[/tex]

Combining the two equations

[tex]\begin{gathered} 9x+7y=37\ldots\ldots\text{.}.1 \\ 3x+5y=17\ldots\ldots2 \end{gathered}[/tex]

Applying the elimination method to resolve the systems of equation

Multiply the second equation by 3, in order to eliminate x

[tex]\begin{gathered} 9x+7y=37\ldots\ldots\ldots1 \\ 3x+5y=17\ldots\ldots\ldots2\times3 \end{gathered}[/tex][tex]\begin{gathered} 9x+7y=37\ldots\ldots\text{.}.1 \\ 9x+15y=51\ldots\ldots2 \end{gathered}[/tex]

Subtract equation 1 from 2

[tex]\begin{gathered} 9x-9x+15y-7y=51-37 \\ 8y=14 \\ \frac{8y}{8}=\frac{14}{8} \\ y=\frac{7}{4}=1.75 \\ \therefore y=1.75 \end{gathered}[/tex]

Substitute y = 1.75 into equation 1 in order to solve for x

[tex]\begin{gathered} 9x+7y=37 \\ 9x+7(1.75)=37 \\ 9x+12.25=37 \\ 9x=37-12.25 \\ 9x=24.75 \\ \frac{9x}{9}=\frac{24.75}{9} \\ x=2.75 \end{gathered}[/tex]

Hence, the cost for each pound of jelly beans = x = $2.75.

the cost for each pound of almonds = y = $1.75.

Three points are shown on the coordinate plane.What is the distance from point A to point B?

Answers

Answer:

The distance from point A to point B is;

[tex]5\text{ units}[/tex]

Explanation:

Given the points A and B with coordinates as shown on the graph;

[tex]\begin{gathered} A(0,5) \\ B(3,1) \end{gathered}[/tex]

Recall that the distance between two points can be calculated using the formula;

[tex]d=\sqrt[]{(x_2-x_1)^2+(y_2-y_1)^2}[/tex]

substituting the coordinates of point A and B. we have;

[tex]\begin{gathered} d=\sqrt[]{(3-0)^2+(1-5)^2} \\ d=\sqrt[]{3^2+4^2} \\ d=\sqrt[]{9+16} \\ d=\sqrt[]{25} \\ d=5 \end{gathered}[/tex]

Therefore, the distance from point A to point B is;

[tex]5\text{ units}[/tex]

Mr.Ortiz has to successfully interview 90% of his assigned households. He was assigned 500 households. He has interviewed 430 households so far. Has he met his goal?

Answers

90 % of 500 is found by

0.9 * 500 = 450

430 < 450

so he has not met his goal

Suppose f(x) = x². Find the graph off(x+3).???

Answers

If f(x)=x^2

Then f(x+3)=(x+3)^2

[tex](x+3)^2=x^2+6x+9[/tex]

Use geogebra to graph the function or calculate the vertex using the equation

x=-b/2a

From the equation we have that

b=6

a=1

x=-6/(2*1)

x=-3

The vertex is on x=-3

Calculate f(-3)=9-18+9=0

The vertex is (-3,0)

y axis cut off point f(0)=0+0+9=9

As "a" is a positive value, parabola open upwards, now you can draw the parabola

This is a sketch, let's use geogebra

Last year, Emma went bowling several times and earned an average score of 130 points. This
year, after taking a class at school, she improved her score to an average of 234 points. What
is the percent of increase in Emma's average score?

Answers

Answer:

80%

Step-by-step explanation:

234-130

=104

(n×p)×100=answer

(130×p)÷100= 10

(130 × p) ÷ 100=104

(130 × p) ÷ 100) × 100 = 104 × 100

130p = 10400

130p ÷ 130 = 10400 ÷ 130

p = 80%

Other Questions
5x + 4y = 12Step 1 of 2: Determine the missing coordinate in the ordered pair (0) so that it will satisfy the given equation, 1 - The Le Mans car race is a 24-hgur race. The longest distance erertraveled by a car in the race is 3,315 miles. What is the distancerounded to the nearest ten?2- The longest river in the world is the Nile. It is 4,184 miles long.What is the length of the Nile River rounded to the nearesthundred miles?3- Frank rounds 846,025 to the nearest hundred thousand and to the nearest ten thousand.Part A What is 846,025 rounded to the nearest hundred thousand?Part B What is 846,025 rounded to the nearest ten thousand?Part C Which rounded number is greater? Explain. A college had 5,000 students in 2018. The number of students decreased by 10% in 2019 andanother 5% in 2020. How many students did the college have in 2020? (1 point) Can you help Turn this equation to the other equation3x-y=-27x+2y=16y=_x+_y=_x+_ Marie requires 2 gallons of paint to cover an area of 400 square feet. Identify the graph that shows the relationship between the quantity of paint and the area covered and the parent function that best describes it. Then use the graph to estimate how many gallons of paint Marie requires to paint an area of 2,400 square feet. please help me I really need help in my math work need helpp pls i don't know how to do it. how do I have to search x in a triangle?? Do anyone know the answer to these questions? Please explain as well Is power based on an election or are people born into power. this is medival europe 7/8 = 7/16 =Reduce your answer to the lowest terms. 1. If triangle ABC is congruent to triangle DEF, DE=17, EF =13, DF =9, and BC = 2x-5, then which of the following is the correctvalue of x?(1) 5(3) 9(2) 7(4) 11 BICPPStatistics: Assignment 2.05Name:We asked 10 randomly selected students in a Speech and Debate class how manycollege courses they had passed prior to taking this class. Their responses areshown below. Find the sample mean and the median.4, 8, 14, 7, 0, 8, 2, 10, 3, 12 012Explanation34BCheck5Use the figure and the table to answer the parts below.67(a) Find the probability that a real number between 4 and 6 is picked.08(b) Find the probability that a real number between 4 and 7 is picked.0RegionABCXArea0.320.560.12 I need help with this math problem 10x + 50 + 6x = 58 if x is the solution to the given equation, what is the value of 32x The student Fun Club plans to go to the movies. At the matinee, tickets cost $6 and popcorn is $3. At evening shows, tickets cost $9 and popcorn is $4. The Fun Club attends a matinee and spends less than $60, and then attends an evening show and spends more than $36. If they purchased the same number of tickets and popcorns at each show, which of the following is a possible solution for the number of tickets and popcorns purchased? in response to a business communication that is vague, what do people tend to do? (choose every correct answer.) The population P of a city is given by P = 115600e^0.024t, where t is the time in years. According to this model, after how many years will the population be 130,000?4.29 years4.89 years5.19 years4.49 years I would like some help to solve this problem number 1 Lucy sold some items at a garage sale. She spent 7/12 of her earnings on a new bike. She uses 3/5 of the remainder to purchase a gift for her mom. What fraction of her total earnings was spent on her mom's gift?